A conjecture about irreducible polynomials with integer coefficients












2












$begingroup$


Let $finmathbb Z[X]$, define $operatorname{P}^+(f)$ as the number of primes $>0$ that $f$ assumes at distinct integral arguments.



Theorem: If $finmathbb Z[X]$ is non constant and reducible of degree $n$, then $operatorname{P}^+(f)leq n$. And for all $n$ there are non constant reducible polynomials of degree n such that $operatorname{P}^+(f)=n$.

[Acta Arith.,104.2 (2002) 117-127.]



Most polynomials are irreducible but using the theorem for an irreducibility test would be inefficient since a lot of irreducible polynomials has a fixed divisor $>1$ and wouldn't pass the test.



A conjecture related to the conjecture of Bunjakowsky states:



Conjecture: If $ finmathbb Z[X]$ is non constant and irreducible, then $f(a)/d$ assumes primes for an infinit number of distinct integral arguments $a$, where $d$ is the largest fixed divisor of $f$.



Irreducibility of Polynomials Whose Coefficients are Integers Page 32.



This makes me wonder if the following hypothesis is true:




$finmathbb Z[X]$ of degree $n>0$ with coprime coefficients is irreducible, iff $;operatorname{P}^+(d^{-1}cdot f)>
n$
or $;operatorname{P}^+(d^{-1}cdot (-f))>
n$
, where $d$ is the greatest fixed divisor of $f$.




$operatorname{P}^+$ is extended above and defined even for integer-valued polynomials with rational coefficients. Proofs or counter-examples?





With a test program using the hypothesis on Eisenstein polynomials $f$ with random coefficients between $-19$ and $19$ and random degree between $2$ and $5$, testing both $f$ and $-f$, resulted in no miss in $1,000,000$ polynomials. The only drawback is the risk of overflow when evaluating the polynomials for higher degrees and greater coefficients.










share|cite|improve this question











$endgroup$












  • $begingroup$
    Bunjakowsky's conjecture starts with $d=1$ where $d = gcd(f(mathbb{Z}))$. What do you get with your question assuming $d=1$ ? If $f(X)= g(X)h(X)$ is reducible and $f(n) = pm p$ then $g(n) = pm 1$ or $h(n) = pm 1$. Write $g(X) = 1+g_2(X)prod_{n in g^{-1}(1)} (X-n)$. What happens if $g(m) = -1$ ? Do you see the problem when $d ne 1$ ?
    $endgroup$
    – reuns
    Jan 20 at 8:43












  • $begingroup$
    @reuns: No! Do you mean that the hypothesis would give false irreducible polynomials?
    $endgroup$
    – Lehs
    Jan 20 at 9:18










  • $begingroup$
    Can you solve the case $d=1$ ? If not then this is what you should ask about. My comment aims at showing how to start with it. If $g(m) = -1$ then $g_2(m)prod_{n in g^{-1}(1)} (m-n) = -2$. Since $2$ is prime that doesn't leave so many choices.
    $endgroup$
    – reuns
    Jan 20 at 9:55










  • $begingroup$
    @reuns: Can you elaborate on this in an answer? It would be most interesting, especially if your idea may be used to construct counter-examples, reducible polynomials that would pass as irreducible.
    $endgroup$
    – Lehs
    Jan 20 at 9:59












  • $begingroup$
    So ? What did you get as improving the simplest case in my answer ?
    $endgroup$
    – reuns
    Jan 22 at 15:40


















2












$begingroup$


Let $finmathbb Z[X]$, define $operatorname{P}^+(f)$ as the number of primes $>0$ that $f$ assumes at distinct integral arguments.



Theorem: If $finmathbb Z[X]$ is non constant and reducible of degree $n$, then $operatorname{P}^+(f)leq n$. And for all $n$ there are non constant reducible polynomials of degree n such that $operatorname{P}^+(f)=n$.

[Acta Arith.,104.2 (2002) 117-127.]



Most polynomials are irreducible but using the theorem for an irreducibility test would be inefficient since a lot of irreducible polynomials has a fixed divisor $>1$ and wouldn't pass the test.



A conjecture related to the conjecture of Bunjakowsky states:



Conjecture: If $ finmathbb Z[X]$ is non constant and irreducible, then $f(a)/d$ assumes primes for an infinit number of distinct integral arguments $a$, where $d$ is the largest fixed divisor of $f$.



Irreducibility of Polynomials Whose Coefficients are Integers Page 32.



This makes me wonder if the following hypothesis is true:




$finmathbb Z[X]$ of degree $n>0$ with coprime coefficients is irreducible, iff $;operatorname{P}^+(d^{-1}cdot f)>
n$
or $;operatorname{P}^+(d^{-1}cdot (-f))>
n$
, where $d$ is the greatest fixed divisor of $f$.




$operatorname{P}^+$ is extended above and defined even for integer-valued polynomials with rational coefficients. Proofs or counter-examples?





With a test program using the hypothesis on Eisenstein polynomials $f$ with random coefficients between $-19$ and $19$ and random degree between $2$ and $5$, testing both $f$ and $-f$, resulted in no miss in $1,000,000$ polynomials. The only drawback is the risk of overflow when evaluating the polynomials for higher degrees and greater coefficients.










share|cite|improve this question











$endgroup$












  • $begingroup$
    Bunjakowsky's conjecture starts with $d=1$ where $d = gcd(f(mathbb{Z}))$. What do you get with your question assuming $d=1$ ? If $f(X)= g(X)h(X)$ is reducible and $f(n) = pm p$ then $g(n) = pm 1$ or $h(n) = pm 1$. Write $g(X) = 1+g_2(X)prod_{n in g^{-1}(1)} (X-n)$. What happens if $g(m) = -1$ ? Do you see the problem when $d ne 1$ ?
    $endgroup$
    – reuns
    Jan 20 at 8:43












  • $begingroup$
    @reuns: No! Do you mean that the hypothesis would give false irreducible polynomials?
    $endgroup$
    – Lehs
    Jan 20 at 9:18










  • $begingroup$
    Can you solve the case $d=1$ ? If not then this is what you should ask about. My comment aims at showing how to start with it. If $g(m) = -1$ then $g_2(m)prod_{n in g^{-1}(1)} (m-n) = -2$. Since $2$ is prime that doesn't leave so many choices.
    $endgroup$
    – reuns
    Jan 20 at 9:55










  • $begingroup$
    @reuns: Can you elaborate on this in an answer? It would be most interesting, especially if your idea may be used to construct counter-examples, reducible polynomials that would pass as irreducible.
    $endgroup$
    – Lehs
    Jan 20 at 9:59












  • $begingroup$
    So ? What did you get as improving the simplest case in my answer ?
    $endgroup$
    – reuns
    Jan 22 at 15:40
















2












2








2





$begingroup$


Let $finmathbb Z[X]$, define $operatorname{P}^+(f)$ as the number of primes $>0$ that $f$ assumes at distinct integral arguments.



Theorem: If $finmathbb Z[X]$ is non constant and reducible of degree $n$, then $operatorname{P}^+(f)leq n$. And for all $n$ there are non constant reducible polynomials of degree n such that $operatorname{P}^+(f)=n$.

[Acta Arith.,104.2 (2002) 117-127.]



Most polynomials are irreducible but using the theorem for an irreducibility test would be inefficient since a lot of irreducible polynomials has a fixed divisor $>1$ and wouldn't pass the test.



A conjecture related to the conjecture of Bunjakowsky states:



Conjecture: If $ finmathbb Z[X]$ is non constant and irreducible, then $f(a)/d$ assumes primes for an infinit number of distinct integral arguments $a$, where $d$ is the largest fixed divisor of $f$.



Irreducibility of Polynomials Whose Coefficients are Integers Page 32.



This makes me wonder if the following hypothesis is true:




$finmathbb Z[X]$ of degree $n>0$ with coprime coefficients is irreducible, iff $;operatorname{P}^+(d^{-1}cdot f)>
n$
or $;operatorname{P}^+(d^{-1}cdot (-f))>
n$
, where $d$ is the greatest fixed divisor of $f$.




$operatorname{P}^+$ is extended above and defined even for integer-valued polynomials with rational coefficients. Proofs or counter-examples?





With a test program using the hypothesis on Eisenstein polynomials $f$ with random coefficients between $-19$ and $19$ and random degree between $2$ and $5$, testing both $f$ and $-f$, resulted in no miss in $1,000,000$ polynomials. The only drawback is the risk of overflow when evaluating the polynomials for higher degrees and greater coefficients.










share|cite|improve this question











$endgroup$




Let $finmathbb Z[X]$, define $operatorname{P}^+(f)$ as the number of primes $>0$ that $f$ assumes at distinct integral arguments.



Theorem: If $finmathbb Z[X]$ is non constant and reducible of degree $n$, then $operatorname{P}^+(f)leq n$. And for all $n$ there are non constant reducible polynomials of degree n such that $operatorname{P}^+(f)=n$.

[Acta Arith.,104.2 (2002) 117-127.]



Most polynomials are irreducible but using the theorem for an irreducibility test would be inefficient since a lot of irreducible polynomials has a fixed divisor $>1$ and wouldn't pass the test.



A conjecture related to the conjecture of Bunjakowsky states:



Conjecture: If $ finmathbb Z[X]$ is non constant and irreducible, then $f(a)/d$ assumes primes for an infinit number of distinct integral arguments $a$, where $d$ is the largest fixed divisor of $f$.



Irreducibility of Polynomials Whose Coefficients are Integers Page 32.



This makes me wonder if the following hypothesis is true:




$finmathbb Z[X]$ of degree $n>0$ with coprime coefficients is irreducible, iff $;operatorname{P}^+(d^{-1}cdot f)>
n$
or $;operatorname{P}^+(d^{-1}cdot (-f))>
n$
, where $d$ is the greatest fixed divisor of $f$.




$operatorname{P}^+$ is extended above and defined even for integer-valued polynomials with rational coefficients. Proofs or counter-examples?





With a test program using the hypothesis on Eisenstein polynomials $f$ with random coefficients between $-19$ and $19$ and random degree between $2$ and $5$, testing both $f$ and $-f$, resulted in no miss in $1,000,000$ polynomials. The only drawback is the risk of overflow when evaluating the polynomials for higher degrees and greater coefficients.







polynomials prime-numbers irreducible-polynomials computational-mathematics conjectures






share|cite|improve this question















share|cite|improve this question













share|cite|improve this question




share|cite|improve this question








edited Jan 23 at 9:08







Lehs

















asked Jan 19 at 14:35









LehsLehs

7,07731662




7,07731662












  • $begingroup$
    Bunjakowsky's conjecture starts with $d=1$ where $d = gcd(f(mathbb{Z}))$. What do you get with your question assuming $d=1$ ? If $f(X)= g(X)h(X)$ is reducible and $f(n) = pm p$ then $g(n) = pm 1$ or $h(n) = pm 1$. Write $g(X) = 1+g_2(X)prod_{n in g^{-1}(1)} (X-n)$. What happens if $g(m) = -1$ ? Do you see the problem when $d ne 1$ ?
    $endgroup$
    – reuns
    Jan 20 at 8:43












  • $begingroup$
    @reuns: No! Do you mean that the hypothesis would give false irreducible polynomials?
    $endgroup$
    – Lehs
    Jan 20 at 9:18










  • $begingroup$
    Can you solve the case $d=1$ ? If not then this is what you should ask about. My comment aims at showing how to start with it. If $g(m) = -1$ then $g_2(m)prod_{n in g^{-1}(1)} (m-n) = -2$. Since $2$ is prime that doesn't leave so many choices.
    $endgroup$
    – reuns
    Jan 20 at 9:55










  • $begingroup$
    @reuns: Can you elaborate on this in an answer? It would be most interesting, especially if your idea may be used to construct counter-examples, reducible polynomials that would pass as irreducible.
    $endgroup$
    – Lehs
    Jan 20 at 9:59












  • $begingroup$
    So ? What did you get as improving the simplest case in my answer ?
    $endgroup$
    – reuns
    Jan 22 at 15:40




















  • $begingroup$
    Bunjakowsky's conjecture starts with $d=1$ where $d = gcd(f(mathbb{Z}))$. What do you get with your question assuming $d=1$ ? If $f(X)= g(X)h(X)$ is reducible and $f(n) = pm p$ then $g(n) = pm 1$ or $h(n) = pm 1$. Write $g(X) = 1+g_2(X)prod_{n in g^{-1}(1)} (X-n)$. What happens if $g(m) = -1$ ? Do you see the problem when $d ne 1$ ?
    $endgroup$
    – reuns
    Jan 20 at 8:43












  • $begingroup$
    @reuns: No! Do you mean that the hypothesis would give false irreducible polynomials?
    $endgroup$
    – Lehs
    Jan 20 at 9:18










  • $begingroup$
    Can you solve the case $d=1$ ? If not then this is what you should ask about. My comment aims at showing how to start with it. If $g(m) = -1$ then $g_2(m)prod_{n in g^{-1}(1)} (m-n) = -2$. Since $2$ is prime that doesn't leave so many choices.
    $endgroup$
    – reuns
    Jan 20 at 9:55










  • $begingroup$
    @reuns: Can you elaborate on this in an answer? It would be most interesting, especially if your idea may be used to construct counter-examples, reducible polynomials that would pass as irreducible.
    $endgroup$
    – Lehs
    Jan 20 at 9:59












  • $begingroup$
    So ? What did you get as improving the simplest case in my answer ?
    $endgroup$
    – reuns
    Jan 22 at 15:40


















$begingroup$
Bunjakowsky's conjecture starts with $d=1$ where $d = gcd(f(mathbb{Z}))$. What do you get with your question assuming $d=1$ ? If $f(X)= g(X)h(X)$ is reducible and $f(n) = pm p$ then $g(n) = pm 1$ or $h(n) = pm 1$. Write $g(X) = 1+g_2(X)prod_{n in g^{-1}(1)} (X-n)$. What happens if $g(m) = -1$ ? Do you see the problem when $d ne 1$ ?
$endgroup$
– reuns
Jan 20 at 8:43






$begingroup$
Bunjakowsky's conjecture starts with $d=1$ where $d = gcd(f(mathbb{Z}))$. What do you get with your question assuming $d=1$ ? If $f(X)= g(X)h(X)$ is reducible and $f(n) = pm p$ then $g(n) = pm 1$ or $h(n) = pm 1$. Write $g(X) = 1+g_2(X)prod_{n in g^{-1}(1)} (X-n)$. What happens if $g(m) = -1$ ? Do you see the problem when $d ne 1$ ?
$endgroup$
– reuns
Jan 20 at 8:43














$begingroup$
@reuns: No! Do you mean that the hypothesis would give false irreducible polynomials?
$endgroup$
– Lehs
Jan 20 at 9:18




$begingroup$
@reuns: No! Do you mean that the hypothesis would give false irreducible polynomials?
$endgroup$
– Lehs
Jan 20 at 9:18












$begingroup$
Can you solve the case $d=1$ ? If not then this is what you should ask about. My comment aims at showing how to start with it. If $g(m) = -1$ then $g_2(m)prod_{n in g^{-1}(1)} (m-n) = -2$. Since $2$ is prime that doesn't leave so many choices.
$endgroup$
– reuns
Jan 20 at 9:55




$begingroup$
Can you solve the case $d=1$ ? If not then this is what you should ask about. My comment aims at showing how to start with it. If $g(m) = -1$ then $g_2(m)prod_{n in g^{-1}(1)} (m-n) = -2$. Since $2$ is prime that doesn't leave so many choices.
$endgroup$
– reuns
Jan 20 at 9:55












$begingroup$
@reuns: Can you elaborate on this in an answer? It would be most interesting, especially if your idea may be used to construct counter-examples, reducible polynomials that would pass as irreducible.
$endgroup$
– Lehs
Jan 20 at 9:59






$begingroup$
@reuns: Can you elaborate on this in an answer? It would be most interesting, especially if your idea may be used to construct counter-examples, reducible polynomials that would pass as irreducible.
$endgroup$
– Lehs
Jan 20 at 9:59














$begingroup$
So ? What did you get as improving the simplest case in my answer ?
$endgroup$
– reuns
Jan 22 at 15:40






$begingroup$
So ? What did you get as improving the simplest case in my answer ?
$endgroup$
– reuns
Jan 22 at 15:40












1 Answer
1






active

oldest

votes


















1












$begingroup$


  • The simplest case :


Let $f in mathbb{Z}[X]$. It is said irreducible iff $f(X) = g(X)h(X) implies g(X)=pm1$ or $h(X) = pm1$. Let $d = gcd(f(mathbb{Z}))$. Assume $d=1$ and $| f(n)|$ is prime more than $2 deg(f)$ times.



If $f(X) = g(X)h(X)$ is reducible, then $f(n) = pm p$ implies
$n$ is a root of $g(X)^2-1$ or $h(X)^2-1$. But those polynomials are of degree $2deg(g), 2 deg(f)-2deg(g)$, so they have at most $2deg(f)$ roots. A contradiction. Whence $f$ is irreducible.




  • To improve the bound $2 deg(f)$, you need to split each case : $g(n)=1,g(n)=-1,h(n)=1,h(n)=-1$ and factorize.


  • Then look at the case $d=2$, you'll have more cases and some of them will probably allow more than $deg(f)$ prime values.







share|cite|improve this answer









$endgroup$













  • $begingroup$
    You are right and I have corrected the question.
    $endgroup$
    – Lehs
    Jan 23 at 9:09











Your Answer





StackExchange.ifUsing("editor", function () {
return StackExchange.using("mathjaxEditing", function () {
StackExchange.MarkdownEditor.creationCallbacks.add(function (editor, postfix) {
StackExchange.mathjaxEditing.prepareWmdForMathJax(editor, postfix, [["$", "$"], ["\\(","\\)"]]);
});
});
}, "mathjax-editing");

StackExchange.ready(function() {
var channelOptions = {
tags: "".split(" "),
id: "69"
};
initTagRenderer("".split(" "), "".split(" "), channelOptions);

StackExchange.using("externalEditor", function() {
// Have to fire editor after snippets, if snippets enabled
if (StackExchange.settings.snippets.snippetsEnabled) {
StackExchange.using("snippets", function() {
createEditor();
});
}
else {
createEditor();
}
});

function createEditor() {
StackExchange.prepareEditor({
heartbeatType: 'answer',
autoActivateHeartbeat: false,
convertImagesToLinks: true,
noModals: true,
showLowRepImageUploadWarning: true,
reputationToPostImages: 10,
bindNavPrevention: true,
postfix: "",
imageUploader: {
brandingHtml: "Powered by u003ca class="icon-imgur-white" href="https://imgur.com/"u003eu003c/au003e",
contentPolicyHtml: "User contributions licensed under u003ca href="https://creativecommons.org/licenses/by-sa/3.0/"u003ecc by-sa 3.0 with attribution requiredu003c/au003e u003ca href="https://stackoverflow.com/legal/content-policy"u003e(content policy)u003c/au003e",
allowUrls: true
},
noCode: true, onDemand: true,
discardSelector: ".discard-answer"
,immediatelyShowMarkdownHelp:true
});


}
});














draft saved

draft discarded


















StackExchange.ready(
function () {
StackExchange.openid.initPostLogin('.new-post-login', 'https%3a%2f%2fmath.stackexchange.com%2fquestions%2f3079407%2fa-conjecture-about-irreducible-polynomials-with-integer-coefficients%23new-answer', 'question_page');
}
);

Post as a guest















Required, but never shown

























1 Answer
1






active

oldest

votes








1 Answer
1






active

oldest

votes









active

oldest

votes






active

oldest

votes









1












$begingroup$


  • The simplest case :


Let $f in mathbb{Z}[X]$. It is said irreducible iff $f(X) = g(X)h(X) implies g(X)=pm1$ or $h(X) = pm1$. Let $d = gcd(f(mathbb{Z}))$. Assume $d=1$ and $| f(n)|$ is prime more than $2 deg(f)$ times.



If $f(X) = g(X)h(X)$ is reducible, then $f(n) = pm p$ implies
$n$ is a root of $g(X)^2-1$ or $h(X)^2-1$. But those polynomials are of degree $2deg(g), 2 deg(f)-2deg(g)$, so they have at most $2deg(f)$ roots. A contradiction. Whence $f$ is irreducible.




  • To improve the bound $2 deg(f)$, you need to split each case : $g(n)=1,g(n)=-1,h(n)=1,h(n)=-1$ and factorize.


  • Then look at the case $d=2$, you'll have more cases and some of them will probably allow more than $deg(f)$ prime values.







share|cite|improve this answer









$endgroup$













  • $begingroup$
    You are right and I have corrected the question.
    $endgroup$
    – Lehs
    Jan 23 at 9:09
















1












$begingroup$


  • The simplest case :


Let $f in mathbb{Z}[X]$. It is said irreducible iff $f(X) = g(X)h(X) implies g(X)=pm1$ or $h(X) = pm1$. Let $d = gcd(f(mathbb{Z}))$. Assume $d=1$ and $| f(n)|$ is prime more than $2 deg(f)$ times.



If $f(X) = g(X)h(X)$ is reducible, then $f(n) = pm p$ implies
$n$ is a root of $g(X)^2-1$ or $h(X)^2-1$. But those polynomials are of degree $2deg(g), 2 deg(f)-2deg(g)$, so they have at most $2deg(f)$ roots. A contradiction. Whence $f$ is irreducible.




  • To improve the bound $2 deg(f)$, you need to split each case : $g(n)=1,g(n)=-1,h(n)=1,h(n)=-1$ and factorize.


  • Then look at the case $d=2$, you'll have more cases and some of them will probably allow more than $deg(f)$ prime values.







share|cite|improve this answer









$endgroup$













  • $begingroup$
    You are right and I have corrected the question.
    $endgroup$
    – Lehs
    Jan 23 at 9:09














1












1








1





$begingroup$


  • The simplest case :


Let $f in mathbb{Z}[X]$. It is said irreducible iff $f(X) = g(X)h(X) implies g(X)=pm1$ or $h(X) = pm1$. Let $d = gcd(f(mathbb{Z}))$. Assume $d=1$ and $| f(n)|$ is prime more than $2 deg(f)$ times.



If $f(X) = g(X)h(X)$ is reducible, then $f(n) = pm p$ implies
$n$ is a root of $g(X)^2-1$ or $h(X)^2-1$. But those polynomials are of degree $2deg(g), 2 deg(f)-2deg(g)$, so they have at most $2deg(f)$ roots. A contradiction. Whence $f$ is irreducible.




  • To improve the bound $2 deg(f)$, you need to split each case : $g(n)=1,g(n)=-1,h(n)=1,h(n)=-1$ and factorize.


  • Then look at the case $d=2$, you'll have more cases and some of them will probably allow more than $deg(f)$ prime values.







share|cite|improve this answer









$endgroup$




  • The simplest case :


Let $f in mathbb{Z}[X]$. It is said irreducible iff $f(X) = g(X)h(X) implies g(X)=pm1$ or $h(X) = pm1$. Let $d = gcd(f(mathbb{Z}))$. Assume $d=1$ and $| f(n)|$ is prime more than $2 deg(f)$ times.



If $f(X) = g(X)h(X)$ is reducible, then $f(n) = pm p$ implies
$n$ is a root of $g(X)^2-1$ or $h(X)^2-1$. But those polynomials are of degree $2deg(g), 2 deg(f)-2deg(g)$, so they have at most $2deg(f)$ roots. A contradiction. Whence $f$ is irreducible.




  • To improve the bound $2 deg(f)$, you need to split each case : $g(n)=1,g(n)=-1,h(n)=1,h(n)=-1$ and factorize.


  • Then look at the case $d=2$, you'll have more cases and some of them will probably allow more than $deg(f)$ prime values.








share|cite|improve this answer












share|cite|improve this answer



share|cite|improve this answer










answered Jan 20 at 10:15









reunsreuns

20.2k21148




20.2k21148












  • $begingroup$
    You are right and I have corrected the question.
    $endgroup$
    – Lehs
    Jan 23 at 9:09


















  • $begingroup$
    You are right and I have corrected the question.
    $endgroup$
    – Lehs
    Jan 23 at 9:09
















$begingroup$
You are right and I have corrected the question.
$endgroup$
– Lehs
Jan 23 at 9:09




$begingroup$
You are right and I have corrected the question.
$endgroup$
– Lehs
Jan 23 at 9:09


















draft saved

draft discarded




















































Thanks for contributing an answer to Mathematics Stack Exchange!


  • Please be sure to answer the question. Provide details and share your research!

But avoid



  • Asking for help, clarification, or responding to other answers.

  • Making statements based on opinion; back them up with references or personal experience.


Use MathJax to format equations. MathJax reference.


To learn more, see our tips on writing great answers.




draft saved


draft discarded














StackExchange.ready(
function () {
StackExchange.openid.initPostLogin('.new-post-login', 'https%3a%2f%2fmath.stackexchange.com%2fquestions%2f3079407%2fa-conjecture-about-irreducible-polynomials-with-integer-coefficients%23new-answer', 'question_page');
}
);

Post as a guest















Required, but never shown





















































Required, but never shown














Required, but never shown












Required, but never shown







Required, but never shown

































Required, but never shown














Required, but never shown












Required, but never shown







Required, but never shown







Popular posts from this blog

Mario Kart Wii

What does “Dominus providebit” mean?

Antonio Litta Visconti Arese